Finding the Domain and Range of a Quadratic Function

  • Thread starter Thread starter n3w ton
  • Start date Start date
  • Tags Tags
    Function
Click For Summary
The discussion centers on finding the domain and range of the quadratic function h(t) = -5t² + 20t, which models the height of a golf ball over time. The domain is determined to be 0 < t < 4 seconds, indicating the time frame in which the ball is in the air. The range is found to be 0 < h < 20 meters, showing the maximum height the ball reaches. Participants suggest graphing the function and using calculus or completing the square to find the maximum height, confirming that the ball will indeed clear the 18-meter tree. Understanding these concepts is essential for solving similar problems in quadratic functions.
n3w ton
Messages
19
Reaction score
0
Question: Ryan is trying to hit a golf ball over a tree that is 18m above the ground. The height of the ball is modeled by the function h(t)= -5t2 + 20t, where t is the time in seconds. Explain the domain and range of the function. Will the ball go over the tree?

This is grade 11 review and I totally forgot how to do this. (I think finding the max/min).

I need some help getting started in the right direction or even finding the answer of the domain (with steps please), and I will try to figure out the range my self.

I already know the answer's I need to know how to get them.

D: (XER | 0 < X <4)
R: (YER | 0 < Y <20)
Meaning the ball will pass over the tree.

But how did they get that! Please help.
 
Physics news on Phys.org
n3w ton said:
Question: Ryan is trying to hit a golf ball over a tree that is 18m above the ground. The height of the ball is modeled by the function h(t)= -5t2 + 20t, where t is the time in seconds. Explain the domain and range of the function. Will the ball go over the tree?

This is grade 11 review and I totally forgot how to do this. (I think finding the max/min).

I need some help getting started in the right direction or even finding the answer of the domain (with steps please), and I will try to figure out the range my self.

I already know the answer's I need to know how to get them.

D: (XER | 0 < X <4)
R: (YER | 0 < Y <20)
Meaning the ball will pass over the tree.

But how did they get that! Please help.

One way to work on this problem is to graph that function, starting at time t=0. Keep plugging in 1 second increments to see what the general shape of the function is.

Have you had some differential calculus? If so, how do you find the max of that function? What can you say about the tangent to the curve at the maximum point of that graph?
 
If you haven't had calculus write it as -5(t2-4t) and complete the square inside the parentheses. It will then be easy to see the range and answer the question.
 
Question: A clock's minute hand has length 4 and its hour hand has length 3. What is the distance between the tips at the moment when it is increasing most rapidly?(Putnam Exam Question) Answer: Making assumption that both the hands moves at constant angular velocities, the answer is ## \sqrt{7} .## But don't you think this assumption is somewhat doubtful and wrong?

Similar threads

Replies
12
Views
2K
  • · Replies 4 ·
Replies
4
Views
2K
  • · Replies 3 ·
Replies
3
Views
2K
  • · Replies 3 ·
Replies
3
Views
1K
  • · Replies 1 ·
Replies
1
Views
2K
  • · Replies 5 ·
Replies
5
Views
2K
  • · Replies 10 ·
Replies
10
Views
2K
  • · Replies 5 ·
Replies
5
Views
2K
  • · Replies 11 ·
Replies
11
Views
3K
  • · Replies 15 ·
Replies
15
Views
2K